Difference between revisions of "2015 AMC 10B Problems/Problem 19"

(Solution)
m (Solution 3)
(33 intermediate revisions by 9 users not shown)
Line 4: Line 4:
 
<math>\textbf{(A) }12+9\sqrt{3}\qquad\textbf{(B) }18+6\sqrt{3}\qquad\textbf{(C) }12+12\sqrt{2}\qquad\textbf{(D) }30\qquad\textbf{(E) }32</math>
 
<math>\textbf{(A) }12+9\sqrt{3}\qquad\textbf{(B) }18+6\sqrt{3}\qquad\textbf{(C) }12+12\sqrt{2}\qquad\textbf{(D) }30\qquad\textbf{(E) }32</math>
  
==Solution==
+
==Solution 1==
The center of the circle lies on the perpendicular bisectors of both chords <math>ZW</math> and <math>YX</math>. Therefore we know the center of the circle must also be the midpoint of the hypotenuse. Let this point be <math>O</math>. Draw perpendiculars to <math>ZW</math> and <math>YX</math> from <math>O</math>, and connect <math>OZ</math> and <math>OY</math>. <math>OY^2=6^2+12^2=180</math>. Let <math>AC=a</math> and <math>BC=b</math>. Then <math>\left(\dfrac{a}{2}\right)^2+\left(a+\dfrac{b}{2}\right)^2=OZ^2=OY^2=180</math>. Simplifying this gives <math>\dfrac{a^2}{4}+\dfrac{b^2}{4}+a^2+ab=180</math>. But by Pythagorean Theorem on <math>\triangle ABC</math>, we know <math>a^2+b^2=144</math>, because <math>AB=12</math>. Thus <math>\dfrac{a^2}{4}+\dfrac{b^2}{4}=\dfrac{144}{4}=36</math>. So our equation simplifies further to <math>a^2+ab=144</math>. However <math>a^2+b^2=144</math>, so <math>a^2+ab=a^2+b^2</math>, which means <math>ab=b^2</math>, or <math>a=b</math>. Aha! This means <math>\triangle ABC</math> is just an isosceles right triangle, so <math>AC=BC=\dfrac{12}{\sqrt{2}}=6\sqrt{2}</math>, and thus the perimeter is <math>\boxed{\textbf{(C)}\ 12+12\sqrt{2}}</math>.
+
The center of the circle lies on the intersection between the perpendicular bisectors of chords <math>ZW</math> and <math>YX</math>. Therefore we know the center of the circle must also be the midpoint of the hypotenuse. Let this point be <math>O</math>. Draw perpendiculars to <math>ZW</math> and <math>YX</math> from <math>O</math>, and connect <math>OZ</math> and <math>OY</math>. <math>OY^2=6^2+12^2=180</math>. Let <math>AC=a</math> and <math>BC=b</math>. Then <math>\left(\dfrac{a}{2}\right)^2+\left(a+\dfrac{b}{2}\right)^2=OZ^2=OY^2=180</math>. Simplifying this gives <math>\dfrac{a^2}{4}+\dfrac{b^2}{4}+a^2+ab=180</math>. But by Pythagorean Theorem on <math>\triangle ABC</math>, we know <math>a^2+b^2=144</math>, because <math>AB=12</math>. Thus <math>\dfrac{a^2}{4}+\dfrac{b^2}{4}=\dfrac{144}{4}=36</math>. So our equation simplifies further to <math>a^2+ab=144</math>. However <math>a^2+b^2=144</math>, so <math>a^2+ab=a^2+b^2</math>, which means <math>ab=b^2</math>, or <math>a=b</math>. <i>Aha</i>! This means <math>\triangle ABC</math> is just an isosceles right triangle, so <math>AC=BC=\dfrac{12}{\sqrt{2}}=6\sqrt{2}</math>, and thus the perimeter is <math>\boxed{\textbf{(C)}\ 12+12\sqrt{2}}</math>.
 
<asy>  
 
<asy>  
 
  /* Geogebra to Asymptote conversion, documentation at artofproblemsolving.com/Wiki, go to User:Azjps/geogebra */
 
  /* Geogebra to Asymptote conversion, documentation at artofproblemsolving.com/Wiki, go to User:Azjps/geogebra */
Line 18: Line 18:
 
draw((3.46,0.96)--(8.02,-3.44)--(12.42,1.12)--(7.86,5.52)--cycle);  
 
draw((3.46,0.96)--(8.02,-3.44)--(12.42,1.12)--(7.86,5.52)--cycle);  
 
  /* draw figures */
 
  /* draw figures */
draw((3.34,-3.86)--(3.14,-3.66)--(3.34,-3.66))
 
 
draw((3.46,0.96)--(3.44,-3.36));  
 
draw((3.46,0.96)--(3.44,-3.36));  
 
draw((3.44,-3.36)--(8.02,-3.44));  
 
draw((3.44,-3.36)--(8.02,-3.44));  
Line 53: Line 52:
 
clip((xmin,ymin)--(xmin,ymax)--(xmax,ymax)--(xmax,ymin)--cycle);  
 
clip((xmin,ymin)--(xmin,ymax)--(xmax,ymax)--(xmax,ymin)--cycle);  
 
</asy>
 
</asy>
 +
 +
==Solution 2==
 +
 +
Let <math>AC = b</math> and <math>BC = a</math> (and we're given that <math>AB=12</math>). Draw line segments <math>YZ</math> and <math>WX</math>. Now we have cyclic quadrilateral <math>WXYZ.</math>
 +
 +
This means that opposite angles sum to <math>180^{\circ}</math>. Therefore, <math>90 + m\angle YZA + 90 - m\angle WXB = 180</math>. Simplifying carefully, we get <math>m\angle YZA = m\angle WXB</math>. Similarly, <math>m\angle{ZYA}</math> = <math>m\angle{XWB}</math>.
 +
 +
That means <math>\triangle ZYA \sim \triangle XWB</math>.
 +
 +
Setting up proportions,
 +
<math>\dfrac{b}{12}=\dfrac{12}{a+b}.</math>
 +
Cross-multiplying we get:
 +
<math>b^2+ab=12^2</math>
 +
 +
But also, by Pythagoras,
 +
<math>b^2+a^2=12^2</math>, so <math>ab=a^2 \Rightarrow a=b</math>
 +
 +
Therefore, <math>\triangle ABC</math> is an isosceles right triangle. <math>AC=BC=\dfrac{12}{\sqrt{2}}=6\sqrt{2}</math>, so the perimeter is <cmath>\boxed{\textbf{(C)}\ 12+12\sqrt{2}}</cmath>
 +
 +
~BakedPotato66
 +
 +
~LegionOfAvatars
 +
 +
==Solution 3==
 +
 +
Both solution 1 and 2 uses Pythagorean Theorem to prove <math>\triangle ABC</math> is isosceles right triangle. I'm going to prove <math>\triangle ABC</math> is isosceles right triangle without using Pythagorean Theorem. I will use geometry rotation.
 +
 +
<asy>
 +
/* Geogebra to Asymptote conversion, documentation at artofproblemsolving.com/Wiki, go to User:Azjps/geogebra */
 +
import graph; size(11.5cm);
 +
real labelscalefactor = 0.5; /* changes label-to-point distance */
 +
pen dps = linewidth(0.7) + fontsize(10); defaultpen(dps); /* default pen style */
 +
pen dotstyle = black; /* point style */
 +
real xmin = -4.3, xmax = 18.7, ymin = -5.26, ymax = 6.3;  /* image dimensions */
 +
 +
draw((3.46,0.96)--(3.44,-3.36)--(8.02,-3.44)--cycle);
 +
draw((3.46,0.96)--(8.02,-3.44)--(12.42,1.12)--(7.86,5.52)--cycle);
 +
/* draw figures */
 +
draw((3.46,0.96)--(3.44,-3.36));
 +
draw((3.44,-3.36)--(8.02,-3.44));
 +
draw((8.02,-3.44)--(3.46,0.96));
 +
draw((3.46,0.96)--(-0.86,0.98));
 +
draw((-0.86,0.98)--(-0.88,-3.34));
 +
draw((-0.88,-3.34)--(3.44,-3.36));
 +
draw((3.46,0.96)--(8.02,-3.44));
 +
draw((8.02,-3.44)--(12.42,1.12));
 +
draw((12.42,1.12)--(7.86,5.52));
 +
draw((7.86,5.52)--(3.46,0.96));
 +
draw((5.74,-1.24)--(-0.86,0.98));
 +
draw((5.74,-1.24)--(-0.87,-1.18), linetype("4 4"));
 +
draw((5.74,-1.24)--(7.86,5.52));
 +
draw((5.74,-1.24)--(10.14,3.32), linetype("4 4"));
 +
draw(shift((5.82,-1.21))*xscale(6.99920709795045)*yscale(6.99920709795045)*arc((0,0),1,19.44457562540183,197.63600413408128), linetype("2 2"));
 +
draw((8.02,-3.44)--(-0.86,0.98));
 +
draw((3.44,-3.36)--(7.86,5.52));
 +
draw((3.44,-3.36)--(5.74,-1.24));
 +
/* dots and labels */
 +
dot((3.46,0.96),dotstyle);
 +
label("$A$", (3.2,1.06), NE * labelscalefactor);
 +
dot((3.44,-3.36),dotstyle);
 +
label("$C$", (3.14,-3.86), NE * labelscalefactor);
 +
dot((8.02,-3.44),dotstyle);
 +
label("$B$", (8.06,-3.8), NE * labelscalefactor);
 +
dot((-0.86,0.98),dotstyle);
 +
label("$Z$", (-1.34,1.12), NE * labelscalefactor);
 +
dot((-0.88,-3.34),dotstyle);
 +
label("$W$", (-1.48,-3.54), NE * labelscalefactor);
 +
dot((12.42,1.12),dotstyle);
 +
label("$X$", (12.5,1.24), NE * labelscalefactor);
 +
dot((7.86,5.52),dotstyle);
 +
label("$Y$", (7.94,5.64), NE * labelscalefactor);
 +
dot((5.74,-1.24),dotstyle);
 +
label("$O$", (5.52,-1.82), NE * labelscalefactor);
 +
clip((xmin,ymin)--(xmin,ymax)--(xmax,ymax)--(xmax,ymin)--cycle);
 +
</asy>
 +
 +
Let <math>O</math> be the the midpoint of <math>AB</math>. The perpendicular bisector of line <math>WZ</math> and <math>XY</math> will meet at <math>O</math>. Thus <math>O</math> is the center of the circle points <math>X</math>, <math>Y</math>, <math>Z</math>, and <math>W</math> lies on.
 +
 +
<math>\angle ZAC=\angle OAY=90^{\circ}</math>, <math>\angle ZAC+\angle BAC=\angle OAY+\angle BAC</math>, <math>\angle ZAB=\angle CAY</math>, and <math>AZ=AC</math>, <math>AB=AY</math>, <math>\triangle AZB \cong \triangle ACY</math> by <math>SAS</math>, <math>BZ=YC</math>. Because <math>AZ \perp AC</math>,  <math>\triangle ACY</math> is a <math>90^{\circ}</math> rotation about point <math>A</math> of <math>\triangle AZB</math>. So, <math>BZ \perp YC</math>.
 +
 +
Because <math>OZ</math> and <math>OY</math> is the radius of <math>\odot O</math>, <math>OZ=OY</math>. Because <math>O</math> is the midpoint of hypotenuse <math>AB</math>, <math>OB=OC</math>, <math>BZ=CY</math>, <math>\triangle OBZ \cong \triangle OCY</math> by <math>SSS</math>. Because <math>BZ \perp CY</math>,  <math>\triangle OCY</math> is a <math>90^{\circ}</math> rotation about point <math>O</math> of <math>\triangle OBZ</math>. So, <math>OB \perp OC</math>.
 +
 +
<math>\angle COB = 90^{\circ}</math>, <math>OC=OB</math>, <math>\triangle OCB</math> is isosceles right triangle, <math>\angle ABC=\angle OBC=45^{\circ}</math>. So, <math>\triangle ABC</math> is isosceles right triangle.
 +
 +
Therefore, <math>AC=BC=\dfrac{12}{\sqrt{2}}=6\sqrt{2}</math>, the perimeter is <math>\boxed{\textbf{(C)}\ 12+12\sqrt{2}}</math>.
 +
 +
~[https://artofproblemsolving.com/wiki/index.php/User:Isabelchen isabelchen]
  
 
==See Also==
 
==See Also==

Revision as of 09:25, 28 December 2021

Problem

In $\triangle{ABC}$, $\angle{C} = 90^{\circ}$ and $AB = 12$. Squares $ABXY$ and $ACWZ$ are constructed outside of the triangle. The points $X, Y, Z$, and $W$ lie on a circle. What is the perimeter of the triangle?

$\textbf{(A) }12+9\sqrt{3}\qquad\textbf{(B) }18+6\sqrt{3}\qquad\textbf{(C) }12+12\sqrt{2}\qquad\textbf{(D) }30\qquad\textbf{(E) }32$

Solution 1

The center of the circle lies on the intersection between the perpendicular bisectors of chords $ZW$ and $YX$. Therefore we know the center of the circle must also be the midpoint of the hypotenuse. Let this point be $O$. Draw perpendiculars to $ZW$ and $YX$ from $O$, and connect $OZ$ and $OY$. $OY^2=6^2+12^2=180$. Let $AC=a$ and $BC=b$. Then $\left(\dfrac{a}{2}\right)^2+\left(a+\dfrac{b}{2}\right)^2=OZ^2=OY^2=180$. Simplifying this gives $\dfrac{a^2}{4}+\dfrac{b^2}{4}+a^2+ab=180$. But by Pythagorean Theorem on $\triangle ABC$, we know $a^2+b^2=144$, because $AB=12$. Thus $\dfrac{a^2}{4}+\dfrac{b^2}{4}=\dfrac{144}{4}=36$. So our equation simplifies further to $a^2+ab=144$. However $a^2+b^2=144$, so $a^2+ab=a^2+b^2$, which means $ab=b^2$, or $a=b$. Aha! This means $\triangle ABC$ is just an isosceles right triangle, so $AC=BC=\dfrac{12}{\sqrt{2}}=6\sqrt{2}$, and thus the perimeter is $\boxed{\textbf{(C)}\ 12+12\sqrt{2}}$. [asy]   /* Geogebra to Asymptote conversion, documentation at artofproblemsolving.com/Wiki, go to User:Azjps/geogebra */ import graph; size(11.5cm);  real labelscalefactor = 0.5; /* changes label-to-point distance */ pen dps = linewidth(0.7) + fontsize(10); defaultpen(dps); /* default pen style */  pen dotstyle = black; /* point style */  real xmin = -4.3, xmax = 18.7, ymin = -5.26, ymax = 6.3;  /* image dimensions */   draw((3.46,0.96)--(3.44,-3.36)--(8.02,-3.44)--cycle);  draw((3.46,0.96)--(8.02,-3.44)--(12.42,1.12)--(7.86,5.52)--cycle);   /* draw figures */ draw((3.46,0.96)--(3.44,-3.36));  draw((3.44,-3.36)--(8.02,-3.44));  draw((8.02,-3.44)--(3.46,0.96));  draw((3.46,0.96)--(-0.86,0.98));  draw((-0.86,0.98)--(-0.88,-3.34));  draw((-0.88,-3.34)--(3.44,-3.36));  draw((3.46,0.96)--(8.02,-3.44));  draw((8.02,-3.44)--(12.42,1.12));  draw((12.42,1.12)--(7.86,5.52));  draw((7.86,5.52)--(3.46,0.96));  draw((5.74,-1.24)--(-0.86,0.98));  draw((5.74,-1.24)--(-0.87,-1.18), linetype("4 4"));  draw((5.74,-1.24)--(7.86,5.52));  draw((5.74,-1.24)--(10.14,3.32), linetype("4 4"));  draw(shift((5.82,-1.21))*xscale(6.99920709795045)*yscale(6.99920709795045)*arc((0,0),1,19.44457562540183,197.63600413408128), linetype("2 2"));   /* dots and labels */ dot((3.46,0.96),dotstyle);  label("$A$", (3.2,1.06), NE * labelscalefactor);  dot((3.44,-3.36),dotstyle);  label("$C$", (3.14,-3.86), NE * labelscalefactor);  dot((8.02,-3.44),dotstyle);  label("$B$", (8.06,-3.8), NE * labelscalefactor);  dot((-0.86,0.98),dotstyle);  label("$Z$", (-1.34,1.12), NE * labelscalefactor);  dot((-0.88,-3.34),dotstyle);  label("$W$", (-1.48,-3.54), NE * labelscalefactor);  dot((12.42,1.12),dotstyle);  label("$X$", (12.5,1.24), NE * labelscalefactor);  dot((7.86,5.52),dotstyle);  label("$Y$", (7.94,5.64), NE * labelscalefactor);  dot((5.74,-1.24),dotstyle);  label("$O$", (5.52,-1.82), NE * labelscalefactor);  clip((xmin,ymin)--(xmin,ymax)--(xmax,ymax)--(xmax,ymin)--cycle);  [/asy]

Solution 2

Let $AC = b$ and $BC = a$ (and we're given that $AB=12$). Draw line segments $YZ$ and $WX$. Now we have cyclic quadrilateral $WXYZ.$

This means that opposite angles sum to $180^{\circ}$. Therefore, $90 + m\angle YZA + 90 - m\angle WXB = 180$. Simplifying carefully, we get $m\angle YZA = m\angle WXB$. Similarly, $m\angle{ZYA}$ = $m\angle{XWB}$.

That means $\triangle ZYA \sim \triangle XWB$.

Setting up proportions, $\dfrac{b}{12}=\dfrac{12}{a+b}.$ Cross-multiplying we get: $b^2+ab=12^2$

But also, by Pythagoras, $b^2+a^2=12^2$, so $ab=a^2 \Rightarrow a=b$

Therefore, $\triangle ABC$ is an isosceles right triangle. $AC=BC=\dfrac{12}{\sqrt{2}}=6\sqrt{2}$, so the perimeter is \[\boxed{\textbf{(C)}\ 12+12\sqrt{2}}\]

~BakedPotato66

~LegionOfAvatars

Solution 3

Both solution 1 and 2 uses Pythagorean Theorem to prove $\triangle ABC$ is isosceles right triangle. I'm going to prove $\triangle ABC$ is isosceles right triangle without using Pythagorean Theorem. I will use geometry rotation.

[asy]   /* Geogebra to Asymptote conversion, documentation at artofproblemsolving.com/Wiki, go to User:Azjps/geogebra */ import graph; size(11.5cm);  real labelscalefactor = 0.5; /* changes label-to-point distance */ pen dps = linewidth(0.7) + fontsize(10); defaultpen(dps); /* default pen style */  pen dotstyle = black; /* point style */  real xmin = -4.3, xmax = 18.7, ymin = -5.26, ymax = 6.3;  /* image dimensions */  draw((3.46,0.96)--(3.44,-3.36)--(8.02,-3.44)--cycle);  draw((3.46,0.96)--(8.02,-3.44)--(12.42,1.12)--(7.86,5.52)--cycle);   /* draw figures */ draw((3.46,0.96)--(3.44,-3.36));  draw((3.44,-3.36)--(8.02,-3.44));  draw((8.02,-3.44)--(3.46,0.96));  draw((3.46,0.96)--(-0.86,0.98));  draw((-0.86,0.98)--(-0.88,-3.34));  draw((-0.88,-3.34)--(3.44,-3.36));  draw((3.46,0.96)--(8.02,-3.44));  draw((8.02,-3.44)--(12.42,1.12));  draw((12.42,1.12)--(7.86,5.52));  draw((7.86,5.52)--(3.46,0.96));  draw((5.74,-1.24)--(-0.86,0.98));  draw((5.74,-1.24)--(-0.87,-1.18), linetype("4 4"));  draw((5.74,-1.24)--(7.86,5.52));  draw((5.74,-1.24)--(10.14,3.32), linetype("4 4"));  draw(shift((5.82,-1.21))*xscale(6.99920709795045)*yscale(6.99920709795045)*arc((0,0),1,19.44457562540183,197.63600413408128), linetype("2 2"));  draw((8.02,-3.44)--(-0.86,0.98));  draw((3.44,-3.36)--(7.86,5.52));  draw((3.44,-3.36)--(5.74,-1.24));   /* dots and labels */ dot((3.46,0.96),dotstyle);  label("$A$", (3.2,1.06), NE * labelscalefactor);  dot((3.44,-3.36),dotstyle);  label("$C$", (3.14,-3.86), NE * labelscalefactor);  dot((8.02,-3.44),dotstyle);  label("$B$", (8.06,-3.8), NE * labelscalefactor);  dot((-0.86,0.98),dotstyle);  label("$Z$", (-1.34,1.12), NE * labelscalefactor);  dot((-0.88,-3.34),dotstyle);  label("$W$", (-1.48,-3.54), NE * labelscalefactor);  dot((12.42,1.12),dotstyle);  label("$X$", (12.5,1.24), NE * labelscalefactor);  dot((7.86,5.52),dotstyle);  label("$Y$", (7.94,5.64), NE * labelscalefactor);  dot((5.74,-1.24),dotstyle);  label("$O$", (5.52,-1.82), NE * labelscalefactor);  clip((xmin,ymin)--(xmin,ymax)--(xmax,ymax)--(xmax,ymin)--cycle);  [/asy]

Let $O$ be the the midpoint of $AB$. The perpendicular bisector of line $WZ$ and $XY$ will meet at $O$. Thus $O$ is the center of the circle points $X$, $Y$, $Z$, and $W$ lies on.

$\angle ZAC=\angle OAY=90^{\circ}$, $\angle ZAC+\angle BAC=\angle OAY+\angle BAC$, $\angle ZAB=\angle CAY$, and $AZ=AC$, $AB=AY$, $\triangle AZB \cong \triangle ACY$ by $SAS$, $BZ=YC$. Because $AZ \perp AC$, $\triangle ACY$ is a $90^{\circ}$ rotation about point $A$ of $\triangle AZB$. So, $BZ \perp YC$.

Because $OZ$ and $OY$ is the radius of $\odot O$, $OZ=OY$. Because $O$ is the midpoint of hypotenuse $AB$, $OB=OC$, $BZ=CY$, $\triangle OBZ \cong \triangle OCY$ by $SSS$. Because $BZ \perp CY$, $\triangle OCY$ is a $90^{\circ}$ rotation about point $O$ of $\triangle OBZ$. So, $OB \perp OC$.

$\angle COB = 90^{\circ}$, $OC=OB$, $\triangle OCB$ is isosceles right triangle, $\angle ABC=\angle OBC=45^{\circ}$. So, $\triangle ABC$ is isosceles right triangle.

Therefore, $AC=BC=\dfrac{12}{\sqrt{2}}=6\sqrt{2}$, the perimeter is $\boxed{\textbf{(C)}\ 12+12\sqrt{2}}$.

~isabelchen

See Also

2015 AMC 10B (ProblemsAnswer KeyResources)
Preceded by
Problem 18
Followed by
Problem 20
1 2 3 4 5 6 7 8 9 10 11 12 13 14 15 16 17 18 19 20 21 22 23 24 25
All AMC 10 Problems and Solutions

The problems on this page are copyrighted by the Mathematical Association of America's American Mathematics Competitions. AMC logo.png